Báo cáo toán học: "The Structure of Maximum Subsets of {1, . . . , n} with No Solutions to a + b = kc" potx

16 268 0
Báo cáo toán học: "The Structure of Maximum Subsets of {1, . . . , n} with No Solutions to a + b = kc" potx

Đang tải... (xem toàn văn)

Tài liệu hạn chế xem trước, để xem đầy đủ mời bạn chọn Tải xuống

Thông tin tài liệu

The Structure of Maximum Subsets of {1, ,n} with No Solutions to a + b = kc Andreas Baltz ∗ Mathematisches Seminar University of Kiel, D-24098 Kiel, Germany aba@numerik.uni-kiel.de Peter Hegarty, Jonas Knape, Urban Larsson Department of Mathematics Chalmers University of Technology G¨oteborg, Sweden hegarty@math.chalmers.se {md9jonas,md0larur}@mdstud.chalmers.se Tomasz Schoen ∗∗ Wydzial Matematyki i Informatyki Adam Mickiewicz University Pozna´n, Poland schoen@amu.edu.pl Submitted: Nov 9, 2004; Accepted: Apr 22, 2005; Published: Apr 28, 2005 MR Subject Classifications: 05D05, 11P99 Abstract If k is a positive integer, we say that a set A of positive integers is k-sum-free if there do not exist a, b, c in A such that a + b = kc. In particular we give a precise characterization of the structure of maximum sized k-sum-free sets in {1, ,n} for k ≥ 4andn large. 1 Introduction A set of positive integers is called k-sum-free if it does not contain elements a, b, c such that a + b = kc, ∗ supported by DFG, Grant SR 7/9 – 2 ∗∗ research partially supported by KBN Grant 2 PO3A 007 24 the electronic journal of combinatorics 12 (2005), #R19 1 where k is a positive integer. Denote by f(n, k) the maximum cardinality of a k-sum-free set in {1, ,n}.Fork = 1 these extremal sets are well-known: Deshoulliers, Freiman, S´os, and Temkin [1] proved in particular that the maximum 1–sum-free sets in {1, ,n} are precisely the set of odd numbers and the “top half” {  n+1 2  , ,n}.Forn>8even { n 2 , ,n− 1} forms the only additional extremal set. The famous theorem of Roth [4] gives f(n, 2) = o(n). Chung and Goldwasser [2] solved the case k = 3 by showing that the set of odd integers is the unique extremal set for n>22. For k ≥ 4theygaveanexample of a k-sum-free set [3] of cardinality k(k−2) k 2 −2 n + 8(k −2) k(k 2 −2)(k 4 −2k 2 −4) n + O(1), which implies lim n→∞ f(n,k) n ≥ k(k−2) k 2 −2 + 8(k −2) k(k 2 −2)(k 4 −2k 2 −4) , and they conjectured that this lower bound is the actual value. Moreover they conjectured that extremal k-sum-free sets consist of three intervals of consecutive integers with slight modifications at the end-points if n is large. In this paper we prove that the first conjecture is true, and we expose a structural result which is very close to the second. Our proof is elementary. In fact it is based on two simple observations: Suppose we are given a k-sum-free set A.Then • kx − y/∈ A for all x, y ∈ A (Otherwise we could satisfy the equation kx =(kx − y)+y in A.) • for all y ∈ A any interval centered around ky 2 cannot share more than half of its elements with A. (Otherwise we would find a pair  ky 2  − d,  ky 2  + d in A, giving  ky 2  − d  +  ky 2  + d  = ky.) 2 Preparations Let n ∈ N be large and let k ∈ N ≥4 . We start by agreeing on some notations. Notations Let A ⊆{1, ,n} be a set of positive integers. Denote by s A := min A and m A := max A the smallest and the largest elements of A respectively. For l, r ∈ R let (l, r]:={x ∈ N | l<x≤ r} [l, r):={x ∈ N | l ≤ x<r} (l, r):={x ∈ N | l<x<r} [l, r]:={x ∈ N | l ≤ x ≤ r} the electronic journal of combinatorics 12 (2005), #R19 2 abbreviate intervals of integers. Continuous intervals will be indicated by the subscript R. Furthermore for any y ∈ N and d ∈ N 0 (:= N ∪{0}) put I d y :=  ky − 1 2 − d, ky +1 2 + d  . Note that if ky is even then I d y =  ky 2 − d, ky 2 − d +1, , ky 2 + d  and |I d y | =2d +1, while if ky is odd we have I d y =  ky−1 2 − d, , ky+1 2 + d  and |I d y | =2d +2. The first Lemma restates our introductory observations. Lemma 1 Let A ⊆ [1,n] be a k-sum-free set. If x, y ∈ A then kx − y/∈ A.Ify ∈ A and d ∈ N 0 then |I d y \ A|≥d +1. Suppose A  is a k-sum-free set consisting of intervals (l i ,r i ]. The interval (l i ,r i ]isk- sum-free if l i ≥ 2r i k . Moreover we observe that reasonably large consecutive intervals (l i+1 ,r i+1 ], (l i ,r i ] (where we assume r i+1 <l i ) should satisfy kr i+1 ≤ l i +s A  . This leads to the following definition, describing a successive transformation of an arbitrary k-sum-free set A into a k-sum-free set of intervals. Definition 1 Let n ∈ N and let A ⊆ [1,n] be k-sum-free with smallest element s := s A . Define sequences (r i ), (l i ), (A i ) by: A 0 := A, r 1 := n, l i :=  2r i k  ,r i+1 :=  l i + s k  , A i := (A i−1 \ (r i+1 ,l i ]) ∪ (l i ,r i ] ∩ [s, n] for i ≥ 1. The letter t = t A will be reserved to denote the least integer such that r t+1 <s. Observe that, for all i ≥ t, A i = A t =[α, r t ] ∪  t−1  j=1 (l j ,r j ]  , (1) where α = α A := max{l t +1,s}. 3 The structure of maximum k-sum-free sets To obtain the structural result we consider the successive transformation of an arbitrary k-sum-free set A intoasetA t of intervals as in (1). Our plan is to show that each member of the transformation sequence (A i )isk-sum-free and has size greater than or equal to |A|.Forn sufficiently large, depending on k, and a maximum sized k-sum-free subset A of [1,n], it will turn out that A t consists of three intervals only, i.e.: that t =3. This observation will do to determine f(n, k), and we conclude our proof by showing that A the electronic journal of combinatorics 12 (2005), #R19 3 could be enlarged if it did not contain (nearly) the whole interval (l 3 ,r 3 ] and consequently almost all elements from (l 2 ,r 2 ]and(l 1 ,r 1 ], so that in fact almost nothing happens during the transformation of an extremal set. Lemma 2 Let A ⊆ [1,n] be k-sum-free. Let i ∈ N. a) A i is k-sum-free. b) |A i |≥|A i−1 |. Proof. a) Clearly, it is enough to prove the claim for i ≤ t, so we may assume that s ≤ r i . Suppose there are a, b, c ∈ A i with a + b = kc. A i is of the form A i = A i−1 ∩ [s, r i+1 ] ∪ (l i ,r i ] ∩ [s, n] ∪ (l i−1 ,r i−1 ] ∪ ∪ (l 1 ,r 1 ]. If c ∈ (l 1 ,r 1 ], then kc > 2n, which is impossible. If i ≥ 2andc ∈ (l j ,r j ] for some j ∈ [2,i], then kc ∈ (2r j ,l j−1 + s] and the larger one of a, b must be in (r j ,l j−1 ]. But (r j ,l j−1 ] ∩ A i = ∅ by construction. Hence c ∈ A i−1 ∩ [s, r i+1 ]. Now, kc ≤ kr i+1 ≤ l i + s. Since (r i+1 ,l i ] ∩ A i = ∅,botha and b have to be in A i−1 ∩ [s, r i+1 ]=A ∩ [s, r i+1 ]. But A is k-sum-free, a contradiction. b) The inequality is trivial for i ≥ t.For1≤ i<twe have that l i ≥ s and hence A i =(A i−1 ∩ [1,r i+1 ]) ∪ (l i ,r i ] ∪  i−1  j=1 (l j ,r j ]  . Thus it suffices to prove that |A i−1 ∩ [1,r i ]|≤|A i−1 ∩ [1,r i+1 ]| +  (k − 2)r i k  . Clearly, then, it suffices to prove the inequality for i = 1, i.e.: to prove that, for any n>0, and any k-sum-free subset A of [1,n] with smallest element s A ,wehave |A|≤|A ∩ [1,r 2,A ]| +  (k − 2)n k  , (2) where r 2,A :=  2n/k + s A k  . The proof is by induction on n. The result is trivial for n = 1. So suppose it holds for all 1 ≤ m<nand let A be a k-sum-free subset of [1,n]. Note that the result is again trivial if s A > 2n/k, so we may assume that s A ≤ 2n/k, which implies that r 2,A ≤ n/k,sincek ≥ 4. First suppose that there exists x ∈ A ∩ (n/k, 2n/k]. Then 1 ≤ kx − n ≤ n and the the electronic journal of combinatorics 12 (2005), #R19 4 map f : y → kx − y is a 1-1 mapping from the interval [kx − n, n] to itself. For each y in this interval, at most one of the numbers y and f(y) can lie in A,sinceA is k-sum-free. To simplify notation, put w := kx − n − 1. Then our conclusion is that |A ∩ (w, n]|≤ 1 2 (n − w). (3) If w =0orifA ∩ [1,w]=∅ , then we are done (since k ≥ 4). Put B := A ∩ [1,w]. Then we may assume B = ∅, hence s B = s A . Applying the induction hypothesis to B, we find that |B| = |A ∩ [1,w]|≤|B ∩ [1,r 2,B ]| +  (k − 2)w k  . (4) But s B = s A implies that r 2,B ≤ r 2,A , hence that B ∩ [1,r 2,B ] ⊆ A ∩ [1,r 2,A ]. Thus (3) and (4) yield the inequality |A|≤|A ∩ [1,r 2,A ]| +  (k − 2)w k  + 1 2 (n − w), which in turn implies (2), since |A| is an integer. Thus we are reduced to completing the induction under the assumption that A ∩ (n/k, 2n/k]=∅. Suppose x ∈ A ∩ (r 2,A ,n/k]. Then 2n/k + s A <kx≤ n and kx − s A ∈ A. In other words, we can pair off elements in A ∩ (r 2,A , 2n/k] with elements in (2n/k, n]\A. This immediately implies (2), and the proofofLemma2iscomplete.  We have seen so far that any k-sum-free set A can be turned into a k-sum-free set A t having overall size at least |A|.ThesetA t is a union of intervals, as given by (1), though note that the final interval [α, r t ] may consist of a single point, since r t = s is possible. The proof of the following Lemma uses a fact shown in [3] by Chung and Goldwasser, to prove that t must be equal to three if |A| is maximum. Lemma 3 Let A be a maximum k-sum-free subset of [1,n], where n>n 0 (k) is sufficiently large. Let s := s A and let t := max{i ∈ N | r i ≥ s}. Then t =3. Proof. Let A t be the set of positive integers given by (1). In a similar manner we now define a k-sum-free subset A  t of (0, 1] . Put c := s/n and, for i =1, , t define real numbers R i ,L i as follows : R 1 := 1,L i := 2R i k ,R i+1 := L i + c k . Then we put A  t := [α  ,R t ) ∪  t−1  j=1 [L j ,R j )  , the electronic journal of combinatorics 12 (2005), #R19 5 where α  := max{L t ,c}.ThatA  t is k-sum-free is shown in [3]. One sees easily that |A t |≤n · µ(A  t )+t, (5) where µ denotes the Lebesgue-measure. Now suppose that t = 3. It is shown in [3] that there exists a constant c k > 0, depending only on k, such that in this case |µ(A  t )|≤ k(k − 2) k 2 − 2 + 8(k − 2) k(k 2 − 2)(k 4 − 2k 2 − 4) − c k . (6) In fact, in the notation of page 8 of [3], an explicit value for c k (which we will use later) is given by c k = 2 k (R(3) − R(4)), which by definition of R amounts to c k = 8(k 4 − 4k 2 − 4)(k − 2) (k 6 − 2k 4 − 4k 2 − 8)(k 4 − 2k 2 − 4)k . (7) Now (5) and (6) would imply that |A|≤ k(k − 2) k 2 − 2 n + 8(k − 2) k(k 2 − 2)(k 4 − 2k 2 − 4) n − c k n + t. But we have seen in the introduction that |A|≥ k(k−2) k 2 −2 n + 8(k −2) k(k 2 −2)(k 4 −2k 2 −4) n + O(1) and, since t = O(log k n), we thus have a contradiction for sufficiently large n. Hence t must equal three, for large enough n, as required.  Now we are nearly in a position to determine f(n, k). We want to calculate the car- dinality of an extremal k-sum-free set A via computing |A 3 |.Since|A 3 | depends on s A , the following lemma will be helpful : Lemma 4 Let n>n 0 (k) be sufficiently large. If A isamaximalk-sum-free subset of [1,n], then S − 2k ≤ s A ≤ S +3, where S :=  8n k 5 −2k 3 −4k . Proof. Set s := s A . By Lemma 3, for n>n 0 (k)wehaver 4 <s.SinceA is maximal we have |A| = |A 3 |. Now, for a fixed n, the cardinality of A 3 is a function of s ∈ [1,n] only. So we need to show that |A 3 (s)| attains its maximum value only for some s ∈ [S − 2k,S +3]. Define s  := min{s ∈ [1,n]:l 3 (s) <s}. A tedious computation (see the Appendix below) yields that s  = S +1ifk is even and s  = S or S +1ifk is odd. Hence s  ∈ [S, S +1]. (8) the electronic journal of combinatorics 12 (2005), #R19 6 Clearly, |A 3 (s)| =   (k−2)n k  + r 2 (s) − l 2 (s)+r 3 (s) − s +1, if s ≥ s  ,  (k−2)n k  + r 2 (s) − l 2 (s)+r 3 (s) − l 3 (s), if s<s  . (9) How does | A 3 (s)| change (ignoring its maximality for a while) if we alter s? First suppose s ≥ s  .Ifs increases by one, then |A 3 | will decrease by one unless either r 2 or r 3 increases. Now r 2 can only increase (by one) once in k(≥ 4) times. Almost the same is true of r 3 , though its dependence on l 2 makes things a little more complicated. However, it is not hard to see that we encounter an irreversible decrease in the cardinality of |A 3 | after at most 3 steps of increment of s. Hence |A 3 (s)| < |A 3 (s  )| if s ≥ s  +3. Next suppose s<s  . If we decrease s,then|A 3 | cannot increase at all, since l i will not decrease unless r i does. Moreover, |A 3 | will become smaller if the size of any interval is di- minished. So we can focus our attention on (l 2 ,r 2 ]. While r 2 decreases once in k times, l 2 does so no more than once in k k 2 ≥2k times. Thus |A 3 (s)| < |A 3 (s  −1)| if s ≤ s  −1−2k. We have now shown that, as a function of s ∈ [1,n], the cardinality of A 3 attains its maximum only for some s ∈ [s  − 2k, s  + 2]. This, together with (8), completes the proof of the lemma.  Now we can prove the first conjecture of Chung and Goldwasser. Theorem 1 lim n→∞ f(n, k) n = k(k − 2) k 2 − 2 + 8(k − 2) k(k 2 − 2)(k 4 − 2k 2 − 4) . Proof. Let A be a maximum k-sum-free set in [1,n], with n sufficiently large. From Lemma 4 we have s A n = S ∗ n + o(1), where S ∗ = 8n k 5 −2k 3 −4k . Thus we can estimate f(n, k) n = |A 3 | n = r 1 − l 1 + r 2 − l 2 + r 3 − S ∗ +1 n + o(1) = 1 n  n − 2n k + 2n + kS ∗ k 2 − 4n +2kS ∗ k 3 + 4n +2kS ∗ + k 3 S ∗ k 4 − S ∗  + o(1) = k 4 − 2k 3 +2k 2 − 4k +4 k 4 + S ∗ nk 3 (2k 2 − 2k +2− k 3 )+o(1) = k 4 − 2k 3 +2k 2 − 4k +4 k 4 + 8(2k 2 − 2k +2− k 3 ) (k 5 − 2k 3 − 4k)k 3 + o(1) = k 5 − 2k 4 − 4k +8 (k 4 − 2k 2 − 4)k + o(1) = k(k − 2) k 2 − 2 + 8(k − 2) k(k 2 − 2)(k 4 − 2k 2 − 4) + o(1), and the claim follows by taking the limit.  We can now show the main result. the electronic journal of combinatorics 12 (2005), #R19 7 Theorem 2 Let k ∈ N ≥4 and n>n 1 (k).LetS and s  be as in Lemma 4. Let A ⊆ {1, ,n} be a k-sum-free set of maximum cardinality, with smallest element s = s A . Then s ∈ [S, S +3]and A = I 3 ∪I 2 ∪I 1 , where I 3 ∈  {[s, r 3 ], [s, r 3 +1]} , if s ≥ s  {[s, r 3 ), [s, r 3 ] \{r 3 − 1}}, if s<s  , I 2 ∈  {[l 2 +2,r 2 ], [l 2 +2,r 2 +1]} , if r 3 +1∈ A {(l 2 ,r 2 ], (l 2 ,r 2 +1], [l 2 ,r 2 ), [l 2 ,r 2 ] \{r 2 − 1}}, if r 3 +1 /∈ A, I 1 ∈  {[l 1 +2,n]}, if r 2 +1∈ A {[l 1 ,n), (l 1 ,n], [l 1 ,n] \{n − 1}}, if r 2 +1 /∈ A, If k is even, then I i =[l i ,r i ] \{r i − 1} for 1 ≤ i ≤ 3. Remark. Note that Theorem 2 does not precisely determine the k-sum-free subsets of {1, , n} of maximum size, for every n>n 1 (k). With n and k fixed, one first needs to determine for which value(s) of s ∈ [S, S +3] the quantity |A 3 (s)|, as given by (9), is maximized. The result will depend on n and k. Even then, for a fixed s, not all the possibilities for I 3 ∪I 2 ∪I 1 need be k-sum-free. See Section 4 below for further discussion. Proof. We have already seen that |A 3 | = |A|. Our first aim is to show by compar- ing A 3 with A 2 that almost the whole interval (l 3 ,r 3 ]mustbeinA. Having achieved this, we infer by Lemma 1 that (r 3 ,l 2 ] ∩ A is nearly empty. Comparing A 2 with A 1 will then reveal that most of (l 2 ,r 2 ] is contained in A. Again Lemma 1 will help us to see that A cannot share many elements with (r 2 ,l 1 ] and a final comparison of A 1 with A will conclude the proof. (I) The first aim is easily reached if s := s A ≥ l 3 + 1. Simply note that A 2 =(A ∩ [s, r 3 ]) ∪ (l 2 ,r 2 ] ∪ (l 1 ,r 1 ] ⊆ [s, r 3 ] ∪ (l 2 ,r 2 ] ∪ (l 1 ,r 1 ]=A 3 . The maximality of |A 2 | gives A 2 = A 3 and hence [s, r 3 ] ⊆ A. Observe that s>l 3 together with Lemma 4 and (8) give S ≤ s ≤ S +3. Assume now that s ≤ l 3 . We want to show that in this case s = l 3 . Suppose s<l 3 and let B =[S − 2k, l 3 ] ∩ A. Define C := I 1 s B ∪  b∈B\{s B } I 0 b . Clearly C ⊆ (l 3 ,r 3 ] for all n  0. Then since C is the union of disjoint intervals, Lemma 1givesthat|C \ A| > |B|. Hence we get the contradiction |A 3 | = |(A 2 \ B) ∪ (l 3 ,r 3 ]|≥ |(A 2 \ B) ∪ (C \ A)| > |A 2 |−|B| + |B| = |A 2 |. Therefore we are left with s = l 3 ,andthis implies |A 2 | = |A 3 |⇐⇒|A ∩ [s, r 3 ]| = |(l 3 ,r 3 ] ∩ [s, r 3 ]| = |(s, r 3 ]|. (10) the electronic journal of combinatorics 12 (2005), #R19 8 If r 3 /∈ A we can infer from (10) that A ∩ [s, r 3 ]=[s, r 3 − 1] = [l 3 ,r 3 − 1]. If r 3 ∈ A, Lemma 1 gives kl 3 − r 3 /∈ A,so−k +1≤ kl 3 − 2r 3 ≤−1. If kl 3 − 2r 3 ≤−2we get I 1 l 3 ⊆ (l 3 ,r 3 ]and|I 1 l 3 \ A|≥2, which is impossible since this would imply |A 3 | > |A 2 |. Hence kl 3 − 2r 3 = −1andk is odd. Using (10) one obtains A ∩ [s, r 3 ]=[l 3 ,r 3 ] \{r 3 − 1}. Suppose now that s = l 3 and r 3 +1∈ A.Thenkl 3 − (r 3 +1)∈ A and r 3 − k ≤ kl 3 − (r 3 +1)≤ r 3 − 1. This contradicts that [s, r 3 − 2] ⊆ A unless kl 3 − (r 3 +1)=r 3 − 1, but then r 3 ∈ A and |A ∩ [s, r 3 ]| = |A ∩ [s, r 3 − 2]| which contradicts (10). Hence r 3 +1 /∈ A if s = l 3 . Finally note that, if s = l 3 and kl 3 ≥ 2r 3 − 1, the latter being a requirement for ei- ther of the two possibilities for I 3 to be k-sum-free, then another computation similar to the one in the Appendix yields that s ≥ S. Again, using Lemma 4 we obtain S ≤ s ≤ S +3, (11) as claimed in the statement of the theorem. This completes the first part of our proof. (II) For the second part note that we have just shown s ≥ l 3 . (12) Plugging (11) into the definition of l 3 yields (after a further tedious computation similar to that in the Appendix) S − 1 ≤ l 3 ≤ S +1, (13) which implies in view of (12) and (11) l 3 ≤ s ≤ l 3 +4. (14) Moreover we have observed that [s, r 3 −2] ⊆ A. Let ξ 1 , ,ξ 5 ∈{0, ,k−1} be constants such that kl 1 =2r 1 − ξ 1 (15) kr 2 = l 1 + s − ξ 2 (16) kl 2 =2r 2 − ξ 3 (17) kr 3 = l 2 + s − ξ 4 (18) kl 3 =2r 3 − ξ 5 . (19) the electronic journal of combinatorics 12 (2005), #R19 9 We suppose that n is sufficiently large, so we can be sure that [ks − (r 3 − 2),k(r 3 − 2) − s] ∩ A = ∅. By (14) we can infer that ∅ =[k(l 3 +4)− (r 3 − 2),k(r 3 − 2) − s] ∩ A =[r 3 − ξ 5 +4k +2,l 2 − ξ 4 − 2k] ∩ A. Let J =[r 3 +2,r 3 − ξ 5 +4k +1]∩ A and K =  x∈J {kx − (s +2),kx− (s +1),kx− s}. Then K ∩ A = ∅, |K| =3|J| and by (18) and (19) we have K ⊆ [l 2 − ξ 4 +2k − 2,l 2 − ξ 4 − kξ 5 +4k 2 + k] ⊆ (l 2 + k − 2,l 2 +4k 2 + k] ⊆ (l 2 +2,r 2 ], if n  0. Let B =[l 2 − ξ 4 − 2k +1,l 2 ] ∩ A.IfB ∪ J ⊆{l 2 } then A ∩ [r 3 +2,l 2 − 1] = ∅. Otherwise, with C as in part (I) if |B| > 1 we can verify that C ⊆ [r 2 − 3k 2 −k+2 2 ,r 2 ] ⊆ (l 2 +1,r 2 ], for n  0, and |C \ A| > |B|.PutC := ∅ if |B|≤1. For large n, K and C are disjoint. Hence |B ∪ J| < |(C \ A) ∪ K| and we get |A 2 | = | [A 1 \ (J ∪ B ∪{r 3 +1})] ∪ (l 2 ,r 2 ]| > |A 1 \{r 3 +1}|. Thus if r 3 +1∈ A we get |A 2 | > |A 1 | so suppose r 3 +1∈ A. Then neither l 2 nor l 2 +1 can be in A 1 . Otherwise, since (s − ξ 4 + k),s− ξ 4 + k − 1 ∈ [s, s + k] ⊆ [s, r 3 − 2] ⊆ A we get k(r 3 +1)=l 2 +(s − ξ 4 + k)=(l 2 +1)+(s − ξ 4 + k − 1), which is impossible. But l 2 +1∈ A 2 , so also in this case it follows that |A 2 | > |A 1 |,since l 2 +1∈ K ∪ C for large n. Again we conclude that A ∩ [r 3 +2,l 2 − 1] = ∅. Consequently, |A 2 | = |A 1 |⇔|A ∩ ([l 2 ,r 2 ] ∪{r 3 +1})| = |(l 2 ,r 2 ]|, which gives A∩ [l 2 ,r 2 ]=[l 2 +2,r 2 ]ifr 3 +1 ∈ A. If r 3 +1 /∈ A and either l 2 /∈ A or r 2 /∈ A, we get A ∩ [l 2 ,r 2 ]=(l 2 ,r 2 ]orA ∩ [l 2 ,r 2 ]=[l 2 ,r 2 ), respectively. In case r 3 +1 /∈ A and both l 2 ,r 2 ∈ A, we see that kl 2 − r 2 = r 2 − ξ 3 /∈ A.Ifξ 3 ≥ 2thenI 1 l 2 ⊆ (l 2 ,r 2 ]andl 2 could be profitably replaced. Hence ξ 3 =1,A ∩ [l 2 ,r 2 ]=[l 2 ,r 2 ] \{r 2 − 1} and k is odd. (III) For the final interval (l 1 ,r 1 ] we use Lemma 1 to conclude from [s, r 3 − 2] ⊆ A and [l 2 +2,r 2 − 2] ⊆ A in view of (16) and (17) that, for n  0, ∅ = A ∩ [k(l 2 +2)− (r 2 − 2),k(r 2 − 2) − (l 2 +2)] = A ∩ [r 2 − ξ 3 +2k +2,l 1 + s − ξ 2 − 2k − l 2 − 2], and ∅ = A ∩ [k(l 2 +2)− (r 3 − 2),k(r 2 − 2) − s] = A ∩ [2r 2 − ξ 3 +2k − r 3 +2,l 1 − ξ 2 − 2k] the electronic journal of combinatorics 12 (2005), #R19 10 [...] .. .< /b> n ], < /b> [s, r3 ] ∪ [l2 + < /b> 1, < /b> r2 + < /b> 1] ∪ [l1 + < /b> 2, < /b> n ], < /b> [s, r3 + < /b> 1] ∪ [l2 + < /b> 2, < /b> r2] ∪ [l1 , < /b> n − 1 ], < /b> [s, r3 + < /b> 1] ∪ [l2 + < /b> 2, < /b> r2] ∪ [l1 + < /b> 1, < /b> n ], < /b> [s, r3 + < /b> 1] ∪ [l2 + < /b> 2, < /b> r2 + < /b> 1] ∪ [l1 + < /b> 2, < /b> n] Note that, by Theorem 2, < /b> for a < /b> given n ≥ 1100 8, < /b> every maximal 4-sum-free subset of < /b> { 1, < /b> < /b> , < /b> n} < /b> is one of < /b> the sets Jx (s ), < /b> for some s ∈ [S, S + < /b> 3] = [s − 1, < /b> s + < /b> 2] By the remarks above, for each i ∈ { 0, < /b> , 109 }, < /b> there are natural .. .< /b> = = = = = = = = = = = = = [S, r3 − 1] ∪ [l2 , < /b> r2 − 1] ∪ [l1 , < /b> n − 1 ], < /b> [S, r3 − 1] ∪ [l2 , < /b> r2 − 1] ∪ [l1 + < /b> 1, < /b> n ], < /b> [S, r3 − 1] ∪ [l2 + < /b> 1, < /b> r2] ∪ [l1 , < /b> n − 1 ], < /b> [S, r3 − 1] ∪ [l2 + < /b> 1, < /b> r2] ∪ [l1 + < /b> 1, < /b> n ], < /b> [S, r3 − 1] ∪ [l2 + < /b> 1, < /b> r2 + < /b> 1] ∪ [l1 + < /b> 2, < /b> n ], < /b> [s, r3 ] ∪ [l2 , < /b> r2 − 1] ∪ [l1 , < /b> n − 1 ], < /b> [s, r3 ] ∪ [l2 , < /b> r2 − 1] ∪ [l1 + < /b> 1, < /b> n ], < /b> [s, r3 ] ∪ [l2 + < /b> 1, < /b> r2 ] ∪ [l1 , < /b> n − 1 ], < /b> [s, r3 ] ∪ [l2 + < /b> 1, < /b> r2 ] ∪ [l1 + < /b> 1 ,. . . 101 and Fi = {J9 (s + < /b> 1)} If |Fi| = 2, < /b> then Fi is {J9 (s ), < /b> J9 (s + < /b> 1)} if i = 9 3, < /b> 10 3, < /b> 10 5, < /b> 10 7, < /b> {J4 , < /b> J9 (s )} if i = 9, < /b> 1 1, < /b> 1 3, < /b> 2 5, < /b> 2 7, < /b> {J8 (s ), < /b> J9 (s )} if i = 4 8, < /b> 5 0, < /b> 5 6, < /b> 5 8, < /b> 6 0, < /b> 7 2, < /b> 7 4, < /b> 7 6, < /b> 8 8, < /b> 90 {J7 (s ), < /b> J9 (s )} if i = 6 3, < /b> 6 5, < /b> 6 7, < /b> 7 9, < /b> 81 If |Fi| = 3: F8 = F24 F15 F29 F39 F62 = F78 F53 F83 F92 F95 = F97 F102 F109 = = = = = = = = = = = {J4 , < /b> J8 (s ), < /b> J9 (s ) }, < /b> {J4 , < /b> J7 (s ), < /b> J9 (s ) }, < /b> {J4 ,. . . ), < /b> J9 (s + < /b> 1) }, < /b> {J9 (s ), < /b> J12 (s ), < /b> J9 (s + < /b> 1) }, < /b> {J6 (s ), < /b> J7 (s ), < /b> J9 (s ) }, < /b> {J9 (s ), < /b> J10 (s ), < /b> J9 (s + < /b> 1) }, < /b> {J7 (s ), < /b> J9 (s ), < /b> J9 (s + < /b> 2) }, < /b> {J8 (s ), < /b> J9 (s ), < /b> J9 (s + < /b> 1) }, < /b> {J7 (s ), < /b> J9 (s ), < /b> J9 (s + < /b> 1) }, < /b> {J9 (s ), < /b> J8 (s + < /b> 1 ), < /b> J9(s + < /b> 1) }, < /b> {J9 (s ), < /b> J7 (s + < /b> 1 ), < /b> J9(s + < /b> 1)} F1 = F3 = F17 F10 = F12 = F26 F38 F41 = F43 F52 = F64 = F66 = F80 F104 = F106 F69 = = = = = = = = {J2 , < /b> J4 , < /b> J7 (s ), < /b> J9 (s )} ,. . . = = = = = {J3 , < /b> J4 , < /b> J6 (s ), < /b> J7 (s ), < /b> J9 (s ) }, < /b> {J2 , < /b> J4 , < /b> J7 (s ), < /b> J9 (s ), < /b> J9 (s + < /b> 2) }, < /b> {J3 , < /b> J4 , < /b> J8 (s ), < /b> J9 (s ), < /b> J9 (s + < /b> 1) }, < /b> {J4 , < /b> J7 (s ), < /b> J9 (s ), < /b> J12 (s ), < /b> J9 (s + < /b> 1) }, < /b> {J6 (s ), < /b> J7 (s ), < /b> J8 (s ), < /b> J9 (s ), < /b> J9 (s + < /b> 2) }, < /b> {J6 (s ), < /b> J7 (s ), < /b> J9 (s ), < /b> J8 (s + < /b> 1 ), < /b> J9(s + < /b> 1) }, < /b> {J7 (s ), < /b> J9 (s ), < /b> J9 (s + < /b> 1 ), < /b> J10 (s + < /b> 1 ), < /b> J9(s + < /b> 2) }, < /b> {J8 (s ), < /b> J9 (s ), < /b> J6 (s + < /b> 1 ), < /b> J7 (s + < /b> 1 ), < /b> J9 (s + < /b> 1)} If |Fi| =. . . F96 = = = = = = {J2 , < /b> J4 , < /b> J7 (s ), < /b> J9 (s ), < /b> J10 (s ), < /b> J9 (s + < /b> 1) }, < /b> {J2 , < /b> J4 , < /b> J7 (s ), < /b> J9 (s ), < /b> J12 (s ), < /b> J9 (s + < /b> 1) }, < /b> {J4 , < /b> J9 (s ), < /b> J12 (s ), < /b> J13 (s ), < /b> J7 (s + < /b> 1 ), < /b> J9 (s + < /b> 1) }, < /b> {J6 (s ), < /b> J7 (s ), < /b> J8 (s ), < /b> J9 (s ), < /b> J10 (s ), < /b> J9 (s + < /b> 1) }, < /b> {J7 (s ), < /b> J9 (s ), < /b> J10 (s ), < /b> J9 (s + < /b> 1 ), < /b> J12 (s + < /b> 1 ), < /b> J9 (s + < /b> 2) }, < /b> {J6 (s ), < /b> J7 (s ), < /b> J8 (s ), < /b> J9 (s ), < /b> J8 (s + < /b> 1 ), < /b> J9 (s + < /b> 1)} If |Fi| = 7: F0 = F16 = {J1 , < /b> J2 ,. . . J4 , < /b> J6 (s ), < /b> J7 (s ), < /b> J8 (s ), < /b> J9 (s ) }, < /b> F40 = {J4 , < /b> J8 (s ), < /b> J9 (s ), < /b> J11 (s ), < /b> J12 (s ), < /b> J8 (s + < /b> 1 ), < /b> J9 (s + < /b> 1)} If |Fi| = 8: F2 F21 F30 F35 F42 F98 = = = = = = {J1 , < /b> J2 , < /b> J3 , < /b> J4 , < /b> J6 (s ), < /b> J7 (s ), < /b> J8 (s ), < /b> J9 (s ) }, < /b> {J2 , < /b> J4 , < /b> J7 (s ), < /b> J9 (s ), < /b> J10 (s ), < /b> J9 (s + < /b> 1 ), < /b> J12 (s + < /b> 1 ), < /b> J9 (s + < /b> 2) }, < /b> {J3 , < /b> J4 , < /b> J6 (s ), < /b> J7 (s ), < /b> J9 (s ), < /b> J12 (s ), < /b> J8 (s + < /b> 1 ), < /b> J9 (s + < /b> 1) }, < /b> {J2 , < /b> J4 , < /b> J7 (s ), < /b> J9 (s ), < /b> J1 2.. < /b> . (s ), < /b> J9 (s + < /b> 1 ), < /b> J10 (s + < /b> 1 ), < /b> J9 (s + < /b> 2) }, < /b> {J3 , < /b> J4 , < /b> J8 (s ), < /b> J9 (s ), < /b> J11 (s ), < /b> J12 (s ), < /b> J8 (s + < /b> 1 ), < /b> J9 (s + < /b> 1) }, < /b> {J6 (s ), < /b> J7 (s ), < /b> J8 (s ), < /b> J9 (s ), < /b> J8 (s + < /b> 1 ), < /b> J9(s + < /b> 1 ), < /b> J10 (s + < /b> 1 ), < /b> J9 (s + < /b> 2)} If |Fi| = 9: F18 = {J1 , < /b> J2 , < /b> J3 , < /b> J4 , < /b> J6 (s ), < /b> J7 (s ), < /b> J8 (s ), < /b> J9 (s ), < /b> J9 (s + < /b> 2) }, < /b> F84 = {J6 (s ), < /b> J7 (s ), < /b> J8 (s ), < /b> J9 (s ), < /b> J10 (s ), < /b> J9 (s + < /b> 1 ), < /b> J12 (s + < /b> 1 ), < /b> J8(s + < /b> 2 ), < /b> J9 (s + < /b> 2)} If |Fi| =. . . (s + < /b> 1) }, < /b> F20 = {J1 , < /b> J2 , < /b> J3 , < /b> J4 , < /b> J6 (s ), < /b> J7 (s ), < /b> J8 (s ), < /b> J9 (s ), < /b> J10 (s ), < /b> J9 (s + < /b> 1 ), < /b> J12 (s + < /b> 1 ), < /b> J8 (s + < /b> 2 ), < /b> J9 (s + < /b> 2) }, < /b> F34 = {J1 , < /b> J2 , < /b> J3 , < /b> J4 , < /b> J6 (s ), < /b> J7 (s ), < /b> J8 (s ), < /b> J9 (s ), < /b> J11 (s ), < /b> J12 (s ), < /b> J8 (s + < /b> 1 ), < /b> J9(s + < /b> 1 ), < /b> J10 (s + < /b> 1 ), < /b> J9 (s + < /b> 2)} Note, in particular, that |F ( 4, < /b> n)| ≤ 14 for all sufficiently large n Computer simulations suggest the same may be true for any even k, with < /b> .. .< /b> correspondences between the sets in the families F ( 4, < /b> n) for all n ≡ i (mod 110) By slight abuse of < /b> notation, we denote any such family simply by Fi Our computer program yielded the following result : If |Fi | = 1, < /b> then i = 6, < /b> 7, < /b> 2 2, < /b> 2 3, < /b> 4 6, < /b> 4 7, < /b> 4 9, < /b> 5 1, < /b> 5 4, < /b> 5 5, < /b> 5 7, < /b> 5 9, < /b> 6 1, < /b> 7 0, < /b> 7 1, < /b> 7 3, < /b> 7 5, < /b> 7 7, < /b> 8 6, < /b> 8 7, < /b> 89 the electronic journal of < /b> combinatorics 12 (2005 ), < /b> #R19 12 or 91 and Fi = {J9 (s ) }, < /b> or i = 3 6, < /b> 3 7, < /b> 100 or . Structure of Maximum Subsets of { 1, ,n} with No Solutions to a + b = kc Andreas Baltz ∗ Mathematisches Seminar University of Kiel, D-24098 Kiel, Germany aba@numerik.uni-kiel.de Peter Hegarty,. be large and let k ∈ N ≥4 . We start by agreeing on some notations. Notations Let A { 1, ,n} be a set of positive integers. Denote by s A := min A and m A := max A the smallest and the largest. (r i+1 ,l i ] ∩ A i = ∅,botha and b have to be in A i−1 ∩ [s, r i+1 ] =A ∩ [s, r i+1 ]. But A is k-sum-free, a contradiction. b) The inequality is trivial for i ≥ t.For1≤ i<twe have that l i ≥

Ngày đăng: 07/08/2014, 08:22

Từ khóa liên quan

Tài liệu cùng người dùng

  • Đang cập nhật ...

Tài liệu liên quan